Ordering four real numbers such that some conditions are satisfied












1












$begingroup$


Suppose I have 4 real numbers in $[0,1]$, all different between each other.



Assume that there exists a way of ordering the four numbers such that:
$$
begin{cases}
w_1>w_2\
w_3=1-w_1\
w_4=1-w_2
end{cases}
$$

where $w_1$ is the first number in the ordered sequence, $w_2$ is the second number in the ordered sequence, $w_3$ is the third number in the ordered sequence, $w_4$ is the fourth number in the ordered sequence.



Is such a way of ordering unique? If yes, could you sketch the proof? In not, could you give a counterexample?










share|cite|improve this question











$endgroup$












  • $begingroup$
    What exactly do you mean when you say unique?
    $endgroup$
    – Ankit Kumar
    Dec 20 '18 at 13:31










  • $begingroup$
    can you give such an assignment to the following four numbers? $frac{1}{4},frac{1}{8},frac{1}{16},frac{1}{32}$
    $endgroup$
    – user408906
    Dec 20 '18 at 13:38










  • $begingroup$
    @Suraj: I'm assuming that such an assignment exists. My assignment requires that you can construct two pairs summing up to 1. In your example, this is not possible.
    $endgroup$
    – STF
    Dec 20 '18 at 13:41






  • 2




    $begingroup$
    What about $0,frac{1}{4},frac{3}{4}, 1$ and $frac{1}{4},1,0,frac{3}{4}$? [I choose to do $w_1<w_2$ and swap $w_3$ and $w_4$ it's the same thing.]
    $endgroup$
    – ancientmathematician
    Dec 20 '18 at 13:52








  • 1




    $begingroup$
    No. Think about it.
    $endgroup$
    – ancientmathematician
    Dec 20 '18 at 13:59
















1












$begingroup$


Suppose I have 4 real numbers in $[0,1]$, all different between each other.



Assume that there exists a way of ordering the four numbers such that:
$$
begin{cases}
w_1>w_2\
w_3=1-w_1\
w_4=1-w_2
end{cases}
$$

where $w_1$ is the first number in the ordered sequence, $w_2$ is the second number in the ordered sequence, $w_3$ is the third number in the ordered sequence, $w_4$ is the fourth number in the ordered sequence.



Is such a way of ordering unique? If yes, could you sketch the proof? In not, could you give a counterexample?










share|cite|improve this question











$endgroup$












  • $begingroup$
    What exactly do you mean when you say unique?
    $endgroup$
    – Ankit Kumar
    Dec 20 '18 at 13:31










  • $begingroup$
    can you give such an assignment to the following four numbers? $frac{1}{4},frac{1}{8},frac{1}{16},frac{1}{32}$
    $endgroup$
    – user408906
    Dec 20 '18 at 13:38










  • $begingroup$
    @Suraj: I'm assuming that such an assignment exists. My assignment requires that you can construct two pairs summing up to 1. In your example, this is not possible.
    $endgroup$
    – STF
    Dec 20 '18 at 13:41






  • 2




    $begingroup$
    What about $0,frac{1}{4},frac{3}{4}, 1$ and $frac{1}{4},1,0,frac{3}{4}$? [I choose to do $w_1<w_2$ and swap $w_3$ and $w_4$ it's the same thing.]
    $endgroup$
    – ancientmathematician
    Dec 20 '18 at 13:52








  • 1




    $begingroup$
    No. Think about it.
    $endgroup$
    – ancientmathematician
    Dec 20 '18 at 13:59














1












1








1


1



$begingroup$


Suppose I have 4 real numbers in $[0,1]$, all different between each other.



Assume that there exists a way of ordering the four numbers such that:
$$
begin{cases}
w_1>w_2\
w_3=1-w_1\
w_4=1-w_2
end{cases}
$$

where $w_1$ is the first number in the ordered sequence, $w_2$ is the second number in the ordered sequence, $w_3$ is the third number in the ordered sequence, $w_4$ is the fourth number in the ordered sequence.



Is such a way of ordering unique? If yes, could you sketch the proof? In not, could you give a counterexample?










share|cite|improve this question











$endgroup$




Suppose I have 4 real numbers in $[0,1]$, all different between each other.



Assume that there exists a way of ordering the four numbers such that:
$$
begin{cases}
w_1>w_2\
w_3=1-w_1\
w_4=1-w_2
end{cases}
$$

where $w_1$ is the first number in the ordered sequence, $w_2$ is the second number in the ordered sequence, $w_3$ is the third number in the ordered sequence, $w_4$ is the fourth number in the ordered sequence.



Is such a way of ordering unique? If yes, could you sketch the proof? In not, could you give a counterexample?







combinatorics permutations combinations






share|cite|improve this question















share|cite|improve this question













share|cite|improve this question




share|cite|improve this question








edited Dec 20 '18 at 13:53







STF

















asked Dec 20 '18 at 13:13









STFSTF

311421




311421












  • $begingroup$
    What exactly do you mean when you say unique?
    $endgroup$
    – Ankit Kumar
    Dec 20 '18 at 13:31










  • $begingroup$
    can you give such an assignment to the following four numbers? $frac{1}{4},frac{1}{8},frac{1}{16},frac{1}{32}$
    $endgroup$
    – user408906
    Dec 20 '18 at 13:38










  • $begingroup$
    @Suraj: I'm assuming that such an assignment exists. My assignment requires that you can construct two pairs summing up to 1. In your example, this is not possible.
    $endgroup$
    – STF
    Dec 20 '18 at 13:41






  • 2




    $begingroup$
    What about $0,frac{1}{4},frac{3}{4}, 1$ and $frac{1}{4},1,0,frac{3}{4}$? [I choose to do $w_1<w_2$ and swap $w_3$ and $w_4$ it's the same thing.]
    $endgroup$
    – ancientmathematician
    Dec 20 '18 at 13:52








  • 1




    $begingroup$
    No. Think about it.
    $endgroup$
    – ancientmathematician
    Dec 20 '18 at 13:59


















  • $begingroup$
    What exactly do you mean when you say unique?
    $endgroup$
    – Ankit Kumar
    Dec 20 '18 at 13:31










  • $begingroup$
    can you give such an assignment to the following four numbers? $frac{1}{4},frac{1}{8},frac{1}{16},frac{1}{32}$
    $endgroup$
    – user408906
    Dec 20 '18 at 13:38










  • $begingroup$
    @Suraj: I'm assuming that such an assignment exists. My assignment requires that you can construct two pairs summing up to 1. In your example, this is not possible.
    $endgroup$
    – STF
    Dec 20 '18 at 13:41






  • 2




    $begingroup$
    What about $0,frac{1}{4},frac{3}{4}, 1$ and $frac{1}{4},1,0,frac{3}{4}$? [I choose to do $w_1<w_2$ and swap $w_3$ and $w_4$ it's the same thing.]
    $endgroup$
    – ancientmathematician
    Dec 20 '18 at 13:52








  • 1




    $begingroup$
    No. Think about it.
    $endgroup$
    – ancientmathematician
    Dec 20 '18 at 13:59
















$begingroup$
What exactly do you mean when you say unique?
$endgroup$
– Ankit Kumar
Dec 20 '18 at 13:31




$begingroup$
What exactly do you mean when you say unique?
$endgroup$
– Ankit Kumar
Dec 20 '18 at 13:31












$begingroup$
can you give such an assignment to the following four numbers? $frac{1}{4},frac{1}{8},frac{1}{16},frac{1}{32}$
$endgroup$
– user408906
Dec 20 '18 at 13:38




$begingroup$
can you give such an assignment to the following four numbers? $frac{1}{4},frac{1}{8},frac{1}{16},frac{1}{32}$
$endgroup$
– user408906
Dec 20 '18 at 13:38












$begingroup$
@Suraj: I'm assuming that such an assignment exists. My assignment requires that you can construct two pairs summing up to 1. In your example, this is not possible.
$endgroup$
– STF
Dec 20 '18 at 13:41




$begingroup$
@Suraj: I'm assuming that such an assignment exists. My assignment requires that you can construct two pairs summing up to 1. In your example, this is not possible.
$endgroup$
– STF
Dec 20 '18 at 13:41




2




2




$begingroup$
What about $0,frac{1}{4},frac{3}{4}, 1$ and $frac{1}{4},1,0,frac{3}{4}$? [I choose to do $w_1<w_2$ and swap $w_3$ and $w_4$ it's the same thing.]
$endgroup$
– ancientmathematician
Dec 20 '18 at 13:52






$begingroup$
What about $0,frac{1}{4},frac{3}{4}, 1$ and $frac{1}{4},1,0,frac{3}{4}$? [I choose to do $w_1<w_2$ and swap $w_3$ and $w_4$ it's the same thing.]
$endgroup$
– ancientmathematician
Dec 20 '18 at 13:52






1




1




$begingroup$
No. Think about it.
$endgroup$
– ancientmathematician
Dec 20 '18 at 13:59




$begingroup$
No. Think about it.
$endgroup$
– ancientmathematician
Dec 20 '18 at 13:59










0






active

oldest

votes











Your Answer





StackExchange.ifUsing("editor", function () {
return StackExchange.using("mathjaxEditing", function () {
StackExchange.MarkdownEditor.creationCallbacks.add(function (editor, postfix) {
StackExchange.mathjaxEditing.prepareWmdForMathJax(editor, postfix, [["$", "$"], ["\\(","\\)"]]);
});
});
}, "mathjax-editing");

StackExchange.ready(function() {
var channelOptions = {
tags: "".split(" "),
id: "69"
};
initTagRenderer("".split(" "), "".split(" "), channelOptions);

StackExchange.using("externalEditor", function() {
// Have to fire editor after snippets, if snippets enabled
if (StackExchange.settings.snippets.snippetsEnabled) {
StackExchange.using("snippets", function() {
createEditor();
});
}
else {
createEditor();
}
});

function createEditor() {
StackExchange.prepareEditor({
heartbeatType: 'answer',
autoActivateHeartbeat: false,
convertImagesToLinks: true,
noModals: true,
showLowRepImageUploadWarning: true,
reputationToPostImages: 10,
bindNavPrevention: true,
postfix: "",
imageUploader: {
brandingHtml: "Powered by u003ca class="icon-imgur-white" href="https://imgur.com/"u003eu003c/au003e",
contentPolicyHtml: "User contributions licensed under u003ca href="https://creativecommons.org/licenses/by-sa/3.0/"u003ecc by-sa 3.0 with attribution requiredu003c/au003e u003ca href="https://stackoverflow.com/legal/content-policy"u003e(content policy)u003c/au003e",
allowUrls: true
},
noCode: true, onDemand: true,
discardSelector: ".discard-answer"
,immediatelyShowMarkdownHelp:true
});


}
});














draft saved

draft discarded


















StackExchange.ready(
function () {
StackExchange.openid.initPostLogin('.new-post-login', 'https%3a%2f%2fmath.stackexchange.com%2fquestions%2f3047530%2fordering-four-real-numbers-such-that-some-conditions-are-satisfied%23new-answer', 'question_page');
}
);

Post as a guest















Required, but never shown

























0






active

oldest

votes








0






active

oldest

votes









active

oldest

votes






active

oldest

votes
















draft saved

draft discarded




















































Thanks for contributing an answer to Mathematics Stack Exchange!


  • Please be sure to answer the question. Provide details and share your research!

But avoid



  • Asking for help, clarification, or responding to other answers.

  • Making statements based on opinion; back them up with references or personal experience.


Use MathJax to format equations. MathJax reference.


To learn more, see our tips on writing great answers.




draft saved


draft discarded














StackExchange.ready(
function () {
StackExchange.openid.initPostLogin('.new-post-login', 'https%3a%2f%2fmath.stackexchange.com%2fquestions%2f3047530%2fordering-four-real-numbers-such-that-some-conditions-are-satisfied%23new-answer', 'question_page');
}
);

Post as a guest















Required, but never shown





















































Required, but never shown














Required, but never shown












Required, but never shown







Required, but never shown

































Required, but never shown














Required, but never shown












Required, but never shown







Required, but never shown







Popular posts from this blog

Quarter-circle Tiles

build a pushdown automaton that recognizes the reverse language of a given pushdown automaton?

Mont Emei